Você está na página 1de 3

MATH 444, Spring 2011

Assignment #7
Solution

1. Let f : R R be continuous at c and let f (c) > 0. Show that there


exists a neighborhood V (c) of c such that if x V (c), then f (x) > 0.
Proof: For = f (c)2
> 0, there exists > such that for all |x c| < ,
i.e., x V (c) = (c , c + ), we have
f (c)
|f (x) f (c)| < = .
2
In this case, we can conclude that
f (c)
0< = f (c)  < f (x).
2

2. Suppose that f : R R is continuous on R and that f (r) = 0 for every


rational number r. Prove that f (x) = 0 for all x R.
Proof: Given any x R, by the density of rational numbers we can
find a sequence of rationals (rn ) such that rn x (how?). Then by the
continuity of f , we have f (x) = lim f (rn ) = 0.

3. Let f : [a, b] R be a continuous function such that f (x) > 0 for each
x [a, b]. Show that there exists a number > 0 such that f (x)
for all x [a, b].
Proof: Since f is continuous on a closed interval [a, b], then there
exists x0 [a, b] such that f (x0 ) f (x) for all x [a, b]. So if we let
= f (x0 ) > 0, then f (x) for all x [a, b].

1
4. Let f : [a, b] R be a continuous function such that for each x [a, b],
there exists y [a, b] such that |f (y)| 21 |f (x)|. Prove that there
exists a point c [a, b] such that f (c) = 0.
Proof: Let us first fix a point x1 [a, b]. Then by the hypothesis,
there exists x2 [a, b] such that |f (x2 )| 21 |f (x1 )|. Apply the hy-
pothesis again, there exists x3 [a, b] such that |f (x3 )| 21 |f (x2 )|
1
22
|f (x1 )|.....
Inductively, we can obtain a sequence of xn [a, b] such that
1 1
|f (xn )| |f (xn1 )| n1 |f (x1 )|.
2 2
Therefore, lim f (xn ) = 0.
On the other hand, since xn is a bounded sequence contained in the
closed interval [a, b], there exists a convergent subsequence (xnk ) such
that xnk c for some c [a, b]. Then by the continuity of f at c, we
have
f (c) = lim f (xnk ) = 0.

1
5. Show that the function f (x) = 1+x2
is uniformly continuous on R.
Proof: Let > 0 be given. q
Without loss of generality, we may assume
that  < 2. Then for M := 2 1 we have

1 1 
< .
1 + x2 1 + M 2 2
Hence, for all |x|, |y| > M , we have

1 1 1 1 2
1 + x2 1 + y 2 1 + x2 + 1 + y 2 < 1 + M 2 < .

Now since f is contionuous on R, it is continuous and thus uniformly


continuous on [M 1, M + 1]. For above > 0, there exists 1 > 0
such that for all x, y [M 1, M + 1], we have |f (x) f (y)| < .
Then for = min{1, 1 } > 0, when x, y R such that |x y| < , we
have
|f (x) f (y)| < .
1
Therefore, f (x) = 1+x2
is uniformly continuous on R.

2
Extra Problems for 4 Hours Credit Students

6. Let f : [a, b] R be a continuous function on [a, b], and assume that


f (a) < 0 < f (b). Let W = {x [a, b] : f (x) < 0}. and let w = sup W .
Prove that f (w) = 0.
Proof: Since f (a) < 0, W is a bounded non-empty subset of [a, b]. We
have w = sup W with x w b for all x W . We want to show that
f (w) = 0. Suppose f (w) 6= 0. Then either f (w) > 0, or f (w) < 0.
If f (w) > 0, then by problem #1 of this assignment, there exists a
neighborhood V (w) = (w , w + ) [a, b] such that f (x) > 0 for all
x V (w). In this case, we must have x w < w if x W , and
thus w is not the least upper bound of W . We get a contradiction.
If f (x) < 0, then by a similar argument, we can find a neighborhood
V (w) = (w , w + ) [a, b] such that f (x) < 0 for all x V (w). In
this case, we have V (w) W and thus w is not an upper bound of W .
We also get a contradiction.
This shows that we must have f (w) = 0.

7. Give an example of a function f : [0, 1] R such that it is discontin-


uous at every point in [0, 1], but |f | is continuous on [0, 1]. (You need
to show that your example satisfies the desired properties)
Answer: Define f (x) on [0, 1] such that f (x) = 1 if x is rational in
[0, 1], and define f (x) = 1 if x is irrantional in [0, 1]. It is clear that
|f (x)| = 1 is continuous on [0, 1].
We claim that f is discontinuous at every point in [0, 1]. We need the
fact that f is discontinuous at c if and only if there exists a sequence
xn c, but lim f (xn ) 6= f (c).
If r is a rational number in [0, 1], then by the density of irrational
numbers, there exists a sequence of irrationals xn r, but lim f (xn ) =
1 6= f (r) = 1. So f is discontinuous at r.
If q is an irrational number in [0, 1], then there exists a sequence of
rationals rn q, but lim f (rn ) = 1 6= f (q) = 1. So f is also discon-
tinuous at q.

Você também pode gostar